Why are integrals well defined?

  • Thread starter Thread starter jamesbob
  • Start date Start date
Click For Summary
Integrals are well-defined when evaluated as improper integrals, particularly when considering limits as the bounds approach infinity. The discussion highlights that the behavior of the integrand at both endpoints is crucial for determining convergence or divergence. For example, the integral of 1/(x + 16)^(5/4) from 0 to infinity needs careful evaluation to ascertain whether it yields a real number. The proper approach involves applying the definition of the improper integral and evaluating limits. Understanding these concepts clarifies why integrals can be considered well-defined in mathematical analysis.
jamesbob
Messages
63
Reaction score
0
Im having trouble explaining why intergrals are well defined. For instance:

\int_{0}^{\infty} \frac{1}{(x + 16)^{\frac{5}{4}}}dx.

Here do i say something like:

\mbox{The integral behaves at zero, and at } \infty, (x + 16)^{\frac{5}{4}} > x^{\frac{5}{4}} \mbox{ therefore the integral diverges.}
 
Last edited:
Physics news on Phys.org
Your sentence is, essentially meaningless:
"The integral behaves at zero" :confused:

Besides, your conclusion is wrong.
 
It depends on what you mean by "well-defined". If you mean "is a real number" then look at what happens when you apply the definition of the improper integral to what you have, and show that you really do get a real number.
 
Actually, the value of your integral equals a most special prime.
 
jamesbob said:
Im having trouble explaining why intergrals are well defined. For instance:

\int_{0}^{\infty} \frac{1}{(x + 16)^{\frac{5}{4}}}dx.

Here do i say something like:

\mbox{The integral behaves at zero, and at } \infty, (x + 16)^{\frac{5}{4}} > x^{\frac{5}{4}} \mbox{ therefore the integral diverges.}
No, it does not make sense. This is an Improper integral. As you know:
\mathop {\int} \limits ^ {b}_{a} f(x) dx = F(b) - F(a), so what if b is not finite, i.e b tends to infinity? We have:
\mathop {\int} \limits ^ {\infty}_{a} f(x) dx = \lim_{b \rightarrow \infty} \mathop {\int} \limits ^ {b}_{a} f(x) dx = \lim_{b \rightarrow \infty}(F(b)) - F(a)
-----------
Say you want to evaluate:
\mathop {\int} \limits ^ {\infty}_{1} \frac{dx}{x ^ 2}
We have:
\mathop {\int} \limits ^ {\infty}_{1} \frac{dx}{x ^ 2} = \left( \lim_{x \rightarrow \infty} - \frac{1}{x} \right) + \frac{1}{1} = 1.
Can you get this? :)
 
Yeah, thanks. Its just the way my silly university asks questions and explains things that confused me. Thanks for the help :smile:
 
Question: A clock's minute hand has length 4 and its hour hand has length 3. What is the distance between the tips at the moment when it is increasing most rapidly?(Putnam Exam Question) Answer: Making assumption that both the hands moves at constant angular velocities, the answer is ## \sqrt{7} .## But don't you think this assumption is somewhat doubtful and wrong?

Similar threads

  • · Replies 6 ·
Replies
6
Views
2K
Replies
4
Views
2K
  • · Replies 5 ·
Replies
5
Views
2K
  • · Replies 6 ·
Replies
6
Views
2K
  • · Replies 9 ·
Replies
9
Views
2K
  • · Replies 8 ·
Replies
8
Views
2K
  • · Replies 47 ·
2
Replies
47
Views
4K
Replies
6
Views
2K
  • · Replies 14 ·
Replies
14
Views
2K
  • · Replies 2 ·
Replies
2
Views
2K